2013 AMC 10B Problems/Problem 16
Revision as of 22:16, 21 February 2013 by Turkeybob777 (talk | contribs) (Fixed LaTex and added second solution)
Problem
In triangle , medians and intersect at , , , and . What is the area of ?
\qquad\textbf{(A)}13\qquad\textbf{(B)}13.5\qquad\textbf{(C)}14\qquad\textbf{(D)}14.5\qquad\textbf{(E)}B1EABA1C1DE2BCABDAPD2AP4AD6CPPEPE=1.5CP=3CE=4.5PED3-4-5DPEAEDCADCE64.5AEDC6*4.5/2=\boxed{\textbf{(B)} 13.5}DPEPP2:1AP=4CP=3AEDC\frac{4\cdot 2+4\cdot 3+3\cdot 2+2\cdot 1.5}{2}=\boxed{\textbf{(B)} 13.5}$